[Math] Showing that the Lebesgue measure is “continuous”

real-analysissolution-verification

Problem: Let $m$ denote the Lebesgue measure on $\mathbb{R}$. Let $E$ be a Lebesgue measurable set of $\mathbb{R}$ such that $m(E \cap (E+t)) = 0$ for all $t \neq 0$. Prove that $m(E)=0$.

I am aware of several solutions to this problem, and some of them we can find here on Math.SE, but I am curious if I can also apply continuity of measure directly.

In particular, we know by the continuity of measure that if $\{B_k\}$ is an ascending sequence of measurable sets, then $m\left( \bigcup^{\infty}_{k=1} B_k\right)=\lim_{k\rightarrow \infty} m(B_k)$.

To specialize this for our case, suppose we consider the sequence $A_n = E \cap (E+\frac{1}{n})$ of sets. Clearly the conditions of the theorem are satisfied, since $m(A_n) < \infty$ and $A_1 \subseteq A_2 \subseteq \cdots$, and so we have that $m\left( \bigcup^{\infty}_{k=1} A_k\right)=\lim_{k\rightarrow \infty} m(A_k) = \lim_{k\rightarrow \infty} m(E \cap (E+\frac{1}{k})) = m(E)$, but this must be zero since unions of sets of measure zero is zero (left-most left-hand side of the equation).

Is this one way to go about this, and if not, where is the fault?

Thank you.

Best Answer

We shall be using the following result:

Lemma. if $E$ is Lebesgue measurable and $m(E)>0$, then for every $\alpha\in (0, 1)$, there is an interval $I$ such that $m(E \cap I) > \alpha \, m(I)$.

Proof. It is a consequence of the Lebesgue Density Theorem, according to which, if $m(E)>0$, then for almost every point $x \in E$ $$ \lim_{\varepsilon\to 0} \frac{m\big(E \cap (x-\varepsilon,x+\varepsilon)\big)}{2\varepsilon} = 1. $$ Hence for such an $x$ we can choose an $\varepsilon$, such that $$ \frac{m\big(E \cap (x-\varepsilon,x+\varepsilon)\big)}{2\varepsilon} > \alpha $$ or equivalently $$ m\big(E \cap (x-\varepsilon,x+\varepsilon)\big)> \alpha\,m\big((x-\varepsilon,x+\varepsilon)\big)\big). \tag*{$\Box$} $$

In our case, as $m(E)>0$, let an interval $I=[a,b]$, such that $m(E\cap I)>\frac{4}{5}m(I)$, and set $t=m(I)/5>0$. Clearly, writing $E_t=E+t$, $$ m\big(E_t\cap [a+t,b]\big)=m\big(E\cap [a,b-t]\big) \ge m(E\cap I)-m\big([b-t,b]\big)\ge \tfrac{3}{5}m(I), $$ and similarly $$ m\big(E\cap [a+t,b-t]\big),\,m\big(E_t\cap [a+t,b-t]\big)\ge\frac{2}{5}m(I), $$ while $m\big([a+t,b-t]\big)=\frac{3}{5}m(I)$. Therefore $$ m(E\cap E_t)\ge m\big(E\cap E_t\cap [a+t,b-t]\big)\ge\frac{1}{5}m(I). $$